What is the Lagrangian, equations of motion for this system?

In summary: Sir, what are the finally correct equations? Are they$$\frac{d}{dt}(\frac{\partial L}{\partial {\dot x}}) - \frac{\partial L}{\partial { x}}=F$$and$$\frac{d}{dt}(\frac{\partial L}{\partial {\dot y}}) - \frac{\partial L}{\partial { y}}=F$$Please clarify thisThe final equations are $$\frac{d}{dt}(\frac{\partial L}{\partial {\dot x}}) - \frac{\partial L}{\partial { x}}=F$$and$$\frac{d}{dt}(\frac{\
  • #1
Amitayas Banerjee
31
2
<<Moderator's note: Moved from a technical forum, no template.>>

FiQkC.jpg

Description of the system:
The masses m1 and m2 lie on a smooth surface. The masses are attached with a spring of non stretched length l0 and spring constant k. A constant force F is being applied to m2.

My coordinates:
Left of m1 there is a stationary wall. The distance between m1's left surface and the wall is represented as y and the distance between the right and left surfaces of m1 and m2, respectively, is represented as x.

What my brain says:
The kinetic energy of this system is equal to the sum of kinetic energies of the two masses, i.e,
$$\frac{1}{2}m_1 \dot{y}^2 + \frac{1}{2}m_2 (\dot{x}+\dot{y})^2$$
and the potential energy is just the spring potential, i.e,
$$\frac{1}{2} k (x-l_0)^2$$
Therefore, the Lagrangian seems to be,
$$L=\frac{1}{2}m_1 \dot{y}^2 + \frac{1}{2}m_2 (\dot{x}+\dot{y})^2 - \frac{1}{2} k (x-l_0)^2$$

Now, the generalized force is
$$F\cdot \frac{\partial {(x+y)}}{\partial x}$$
So, the equations of motion seem to be,
$$\frac{d}{dt}(\frac{\partial L}{\partial {\dot x}}) - \frac{\partial L}{\partial { x}}=F$$
and
$$\frac{d}{dt}(\frac{\partial L}{\partial {\dot y}}) - \frac{\partial L}{\partial { y}}=0$$Is this correct? Or am I missing something? Please help me figure out the mistakes.
 

Attachments

  • FiQkC.jpg
    FiQkC.jpg
    7.5 KB · Views: 791
Last edited by a moderator:
Physics news on Phys.org
  • #2
Your coordinates are set up in such a manner that the displacement depends on both x and y.

Also note that you perfectly well can treat the force using a potential, since it is conservative.
 
  • #3
Orodruin said:
Your coordinates are set up in such a manner that the displacement depends on both x and y.
Sir, What is the problem if the displacement of m2 depends on both x and y? How can I modify my equations for that? Is there any problem with my generalized force calculation?
 
Last edited:
  • Like
Likes Delta2
  • #4
The generalised force for any coordinate is given by ##F(\partial \ell/dq^i)##, where ##\ell## is the displacement and ##F## the force. You have ignored the effect of the generalised force on the ##y## coordinate.
 
  • #5
Orodruin said:
You have ignored the effect of the generalised force on the yyy coordinate.
Is it not $$F\cdot \frac{\partial {(x+y)}}{\partial x}$$
The factor of F becomes 1. Does it not?
 
  • #6
Amitayas Banerjee said:
Is it not $$F\cdot \frac{\partial {(x+y)}}{\partial x}$$
The factor of F becomes 1. Does it not?
That is just the effect on the ##x## coordinate, but you are ignoring the fact that ##x + y## also depends on ##y##.
 
  • #7
Orodruin said:
That is just the effect on the xxx coordinate, but you are ignoring the fact that x+yx+yx + y also depends on yyy.
Then my second equation of motion is $$
\frac{d}{dt}(\frac{\partial L}{\partial {\dot y}}) - \frac{\partial L}{\partial { y}}=F\cdot \frac{\partial {(x+y)}}{\partial y}$$? Is it correct now?
 
  • #8
Yes. However, as I said, it is easier to do this by including a potential in the Lagrangian.
 
  • #9
Sir, what are the finally correct equations? Are they
$$\frac{d}{dt}(\frac{\partial L}{\partial {\dot x}}) - \frac{\partial L}{\partial { x}}=F$$

and

$$\frac{d}{dt}(\frac{\partial L}{\partial {\dot y}}) - \frac{\partial L}{\partial { y}}=F$$
? Please clarify this
 
Last edited:
  • Like
Likes Delta2
  • #10
I think they are correct now, probably Orodruin wants to incorporate the external force into the Lagrangian by adding a term -F(x+y) as extra potential energy to the lagrangian, so it will be ##L'=L-F(x+y)## and the equations of motion with ##L'## will be homogeneous.
 

Related to What is the Lagrangian, equations of motion for this system?

What is the Lagrangian?

The Lagrangian is a mathematical function that describes the dynamics of a physical system. It is commonly used in classical mechanics to calculate the equations of motion for a system.

What are equations of motion?

Equations of motion are mathematical equations that describe the motion of a system over time. They are derived from the Lagrangian and are used to predict the future behavior of a system.

How is the Lagrangian derived?

The Lagrangian is derived using the principle of least action, which states that the path a system takes between two points in time is the one that minimizes the total action. The action is defined as the integral of the Lagrangian over time.

What is the importance of the Lagrangian in physics?

The Lagrangian is important because it provides a more elegant and intuitive way to describe the dynamics of a system compared to traditional Newtonian mechanics. It also allows for the incorporation of constraints and generalized coordinates, making it applicable to a wide range of physical systems.

Can the Lagrangian be used for all systems?

The Lagrangian can be used for most mechanical systems, including those with non-conservative forces and constraints. However, it may not be applicable to systems with quantum effects or relativistic effects, in which case other mathematical approaches must be used.

Similar threads

  • Advanced Physics Homework Help
Replies
11
Views
1K
  • Advanced Physics Homework Help
Replies
2
Views
842
  • Advanced Physics Homework Help
Replies
1
Views
217
  • Advanced Physics Homework Help
Replies
2
Views
518
Replies
7
Views
2K
  • Advanced Physics Homework Help
Replies
5
Views
1K
Replies
6
Views
993
  • Advanced Physics Homework Help
Replies
16
Views
931
  • Advanced Physics Homework Help
Replies
0
Views
685
  • Advanced Physics Homework Help
Replies
26
Views
2K
Back
Top